Archer Review 9b

Réussis tes devoirs et examens dès maintenant avec Quizwiz!

Which risk factor relating to fluid and electrolyte imbalances is accurately paired with its specific fluid or electrolyte disorder? Select all that apply. A. Hypermagnesemia: Alcoholism [7%] B. Hypomagnesemia: Cushing's syndrome [6%] C. Hyperkalemia: Renal failure [23%] D. Hypokalemia: Hypomagnesemia [7%] E. Hypernatremia: Dehydration [20%] F. Hyponatremia: Diarrhea [14%] G. Hypercalcemia: Hypoparathyroidism [10%] H. Hypocalcemia: Hypomagnesemia [12%]

Explanation Choices C, D, E, F, and H are correct. The risk factors/causes of fluid and electrolyte imbalances that are paired accurately are: Hyperkalemia: Renal failure (Choice C) is correct. Hyperkalemia is a common finding seen in renal failure and may be life-threatening. Adequate kidney function is necessary to excrete potassium. End-stage renal disease (ESRD) is often characterized by hyperkalemia, fluid overload, metabolic acidosis, and pulmonary congestion. Dialysis is indicated in ESRD to eliminate excess fluid, excess acid, and excess potassium. Hypokalemia: Hypomagnesemia (Choice D) and Hypocalcemia: Hypomagnesemia (Choice H) are correct. Hypomagnesemia is often associated with hypokalemia and hypocalcemia. Untreated hypomagnesemia may lead to treatment-refractory hypokalemia. The mechanism for hypomagnesemia-induced hypokalemia involves increased urinary potassium wasting via collecting duct cells due to a reduction in intracellular magnesium. Hypomagnesemia can also cause hypocalcemia. Magnesium is necessary for the production and release of parathyroid hormone. Magnesium deficiency results in decreased production of parathyroid hormone and consequently, secondary hypocalcemia. It is important to always look for and correct hypomagnesemia when treating hypokalemia, and hypocalcemia. Hypernatremia: Dehydration (Choice E) is correct. Dehydration results in loss of free water more than the sodium and often, causes hypovolemic hypernatremia. Hypernatremia increases serum osmolality. Increased osmolality triggers Anti-Diuretic Hormone (ADH) release. ADH helps the body to conserve water and concentrates the urine. Hyponatremia: Chronic Diarrhea (Choice F) is correct. Hypovolemic hyponatremia can occur with chronic diarrhea or persistent nausea/ vomiting. Chronic diarrhea results in loss of sodium and water. Ensuing decreased intravascular fluid volume triggers ADH release. ADH causes kidneys to retain free water further aggravating hyponatremia. Choice A is incorrect. Alcoholism leads to hypomagnesemia, not hypermagnesemia. Hypermagnesemia is an uncommon electrolyte imbalance. Risk factors/ causes of hypermagnesemia include renal failure, adrenal insufficiency (Addison's disease), hypothyroidism, drugs like Lithium, and increased intake of magnesium salts/ magnesium-containing laxatives. Choice B is incorrect. A risk factor that is associated with hypomagnesemia is alcoholism, not Cushing's syndrome. Other risk factors of hypomagnesemia include refeeding syndrome, diuretic use, and proton pump inhibitor (PPI) use. Choice G is incorrect. Hypoparathyroidism causes hypocalcemia, not hypercalcemia. Decreased parathyroid hormone (PTH) results in decreased calcium and increased phosphorous. Some risk factors/ causes associated with hypercalcemia include hyperparathyroidism, thiazide diuretics, malignancy (myeloma, metastatic cancer), milk-alkali syndrome, vitamin d toxicity, and sarcoidosis.

The nurse is caring for a client with the below tracing on the electrocardiogram (ECG). The nurse should anticipate which prescription from the primary healthcare provider (PHCP)? See the exhibit. A. captopril [9%] B. atropine [19%] C. adenosine [35%] D. diltiazem [37%]

Explanation Choice D is correct. The tracing in the exhibit shows irregularly irregular rhythm with no identifiable p-waves. This reflects atrial fibrillation. Diltiazem is a calcium channel blocker and provides rate control in atrial fibrillation. Atrial fibrillation results in increased ventricular rate and reduced ventricular diastolic filling. If the ventricular rate is not controlled, cardiac output is reduced, resulting in hypotension and congestive heart failure. Initial treatment in atrial fibrillation is aimed at ventricular rate control with calcium channel blockers (diltiazem, verapamil, amiodarone), a beta-blocker ( atenolol, metoprolol), or digoxin. If the atrial fibrillation remains persistent, cardioversion is considered. Choices A, B, and C are incorrect. Captopril ( Choice A) is an ACE inhibitor used to treat heart failure and hypertension. Atropine ( Choice B) increases the heart rate and is efficacious for symptomatic sinus bradycardia, not in atrial fibrillation. Adenosine ( Choice C) is approved for supraventricular tachycardia when vagal maneuvers are not efficacious. Note that the term " supraventricular tachycardia ( SVT)" refers to a wide variety of atrial arrhythmias ( atrial flutter, atrial fibrillation, atrial tachycardia) when the rhythm can not be clearly identified. During an SVT, the heart rate is very high at 150 to 220 beats per minute. The rate needs to be slowed down so the rhythm can be identified properly and treated specifically. Vagal maneuvers ( carotid sinus massage, Valsalva maneuver) are applied first. If the SVT is refractory to vagal maneuvers, IV adenosine is used to slow down or terminate it. Because the rhythm strip in the exhibit clearly shows fibrillatory waves ( atrial fibrillation), adenosine is not necessary and must proceed with treating the rhythm as atrial fibrillation. Learning Objective Recognize that controlling the ventricular rate is an immediate goal of treating atrial fibrillation. Calcium channel blockers and beta-blockers are the most common agents used for rate control. Additional Info Atrial fibrillation is associated with atrial fibrosis and loss of atrial muscle mass. These structural changes are common in heart diseases such as hypertension, heart failure, and coronary artery disease. Characteristically, atrial fibrillation is irregularly, irregular with no P-waves identified. The most significant complication associated with atrial fibrillation is embolic stroke. Other complications include congestive heart failure and myocardial infarction. Atrial fibrillation promotes blood pooling ( stasis) in the atrium and facilitates clot formation ( thrombus). Arterial thromboembolism may eventually follow, leading to ischemic stroke. Treatment options for atrial fibrillation are aimed at rate control and anticoagulation. Digoxin (not as commonly used), amiodarone, diltiazem, verapamil, or atenolol are the medications used to achieve rate control in atrial fibrillation. The client may be prescribed an oral anticoagulant ( apixaban, rivaroxaban, dabigatran, or warfarin) to prevent thrombosis/ stroke. If medication is not desired, synchronized cardioversion may be prescribed.

Which aspect of the therapeutic communication process is the most ambiguous and the most often misinterpreted by the receiver of the message? A. A facial grimace or some form of body posturing by the sender of the oral message. [43%] B. Sending messages with medical jargon that are not understandable to the client. [41%] C. Facilitating the client to reflect on the statement that they have just stated. [5%] D. Probing the client as the sender of the message in an intrusive and invasive manner.

Explanation Choice A is correct. A facial grimace or some form of body posturing by the sender of the verbal message is the most ambiguous and the most often misinterpreted by the receiver of the word. Some nonverbal cues, such as facial grimace and body posturing, are highly vague and not at all with a clear meaning. This lack of clarity of the nonverbal message often leads to misinterpretation by the receiver of the word. Choice B is incorrect. Although sending messages with medical jargon is not understandable to the client and is not acceptable nursing practice. Choice C is incorrect. Facilitating the client to reflect on a statement is a useful therapeutic communication technique that should be used. This technique does not lead to ambiguity or misinterpretation. Choice D is incorrect. Although probing the client as the sender of the message in an intrusive and invasive manner is not therapeutic, it does not lead to ambiguity or misinterpretation.

The nurse is assessing patients requesting oral contraceptives in the gynecology clinic. Which patient will the nurse recommend to the physician to be prescribed oral contraceptives? A. A 29-year-old woman with a history of abortion [66%] B. A 32-year-old woman with a history of deep vein thrombosis (DVT) [18%] C. A 34-year-old whose father died of a cerebrovascular accident [7%] D. A 41-year-old woman who smokes [9%]

Explanation Choice A is correct. A history of abortion is not a contraindication for oral contraceptives. Choice B is incorrect. Oral contraceptives are contraindicated in women with a history of deep vein thrombosis (DVT). A DCT indicates a history of increased clotting, and this coagulopathy can be increased if the patient is prescribed contraceptives that contain estrogen. Choice C is incorrect. Oral contraceptives are contraindicated in women with a family history of cerebrovascular accidents or myocardial infarction because of their increased tendency toward clotting as an effect of increased estrogen. Choice D is incorrect. It is contraindicated for women who smoke and are over 40-years-old to take oral contraceptives.

Which of the following nursing leadership and management-related statements is accurate? A. Nurse managers possess personal accountability for not only their own specific acts but also acts of delegation and supervision. [67%] B. Nurses and nurse managers are accountable for supervising others, except for unlicensed assistive personnel (UAP). [17%] C. When delegated to the appropriate person, the delegating nurse is no longer responsible for ensuring the task is performed correctly. [2%] D. Managing care entails micromanagement of the unit's staff. [13%]

Explanation Choice A is correct. Accountability cannot be delegated. The delegating nurse is ultimately responsible for ensuring the task is performed correctly. When entrusting the care of clients to others, you are able to transfer the responsibility and authority to do the job while retaining the liability for the quality of care provided to the client(s). Therefore, Choice A is correct. Choice B is incorrect. Nurses and nurse managers retain accountability for the supervision of others, including those who are often unlicensed assistive personnel (UAP). Moreover, supervision includes monitoring the tasks performed, ensuring that functions are performed in an appropriate fashion, and ensuring that assigned tasks and functions do not exceed competency or require a license to perform. Therefore, Choice B is incorrect. Choice C is incorrect. No matter to whom a task is delegated, the delegating nurse will remain responsible for ensuring that the task is performed correctly. Therefore, Choice C is incorrect. Choice D is incorrect. Micromanagement is not only unnecessary, but it harbors an unfavorable work environment and decreases staff confidence levels. Effective leaders are confident in their staff and the training they have provided to their staff members, utilizing the appropriate periodic assessment methods to ensure quality. Therefore, Choice D is incorrect. Learning Objective Identify the correct statement related to nursing leadership and management. Additional Info The delegatee must have been trained and validated as competent to perform the task or responsibility. The nurse cannot delegate a task that requires nursing judgment or critical thinking. Delegation can only occur by an individual that owns delegating authority. The delegating nurse is ultimately responsible for ensuring the task is performed correctly. According to the nurse practice act, any delegated task must be within the scope of practice of the delegator and the delegatee. Within every health care setting, the delegation process is multidimensional, beginning at the administrative level. Administrators are responsible for determining the criteria and creating the policies and procedures governing the delegation of nursing responsibilities. Administrators are also responsible for the periodic assessment of the delegation process and the ongoing promotion of a healthy working environment. In 2019, the NCSBN published national guidelines outlining the five rights of delegation. These rights included the following: (1) Delegation of the right task (2) Delegation under the right conditions (3) Delegation to the right individual (4) Delegation by providing the right instructions and effective communication (5) Delegation with the right supervision and assessment

This nurse is caring for a client who is receiving prescribed aripiprazole. Which of the following findings would indicate a therapeutic response? A. Reports of no hallucinations and delusions [44%] B. Increased concentration and attention [19%] C. Improved muscle coordination and gait [25%] D. No reports of insomnia or night terrors [13%]

Explanation Choice A is correct. Aripiprazole is an atypical (second generation) antipsychotic indicated in treating schizophrenia and certain mood disorders such as bipolar. If the client reports no hallucinations or delusions, a positive symptom associated with schizophrenia, this medication has exerted its therapeutic effect. Choices B, C, and D are incorrect. Aripiprazole is not indicated in the treatment of attention deficit and hyperactivity disorder (ADHD). Further, this medication would not improve muscle coordination or gait because, adversely, it may cause movement disorders such as dystonia. Finally, this medication is not intended to decrease reports of insomnia or night terrors. Aripiprazole has a low sedation effect and would not be beneficial for those with insomnia. Additional Info Aripiprazole is a second-generation (atypical) antipsychotic. Other medications in this class include ziprasidone, risperidone, olanzapine, and clozapine. Aripiprazole is a novel antipsychotic implicated in causing less weight gain and metabolic problems compared to the other atypical antipsychotics. This medication is approved to treat schizophrenia and as an adjunct for those with major depressive disorder.

The RN is caring for a 72-year-old patient on the medical-surgical floor. Which of the following factors would not be an indication for this patient to receive parenteral nutrition? A. Dysphagia [38%] B. Gastrointestinal obstruction [26%] C. Severe anorexia nervosa [22%] D. Severe burns [14%]

Explanation Choice A is correct. Dysphagia indicates an inability to swallow. Parenteral nutrition aims to meet the body's need for nutrients via a route other than the GI tract (i.e. bloodstream) and is indicated when the GI tract is unable to ingest, digest, or absorb nutrients. A patient who is unable to swallow may require an alternate form of nutrient delivery, such as enteral nutrition, but this would not necessarily be an indication for parenteral nutrition. Choice B is incorrect. A gastrointestinal obstruction would interfere with the GI tract's ability to ingest, digest, and absorb nutrients, so this is a common indication for parenteral nutrition. Choice C is incorrect. Severe anorexia nervosa would interfere with the GI tract's ability to ingest, digest, and absorb nutrients, so this is a common indication for parenteral nutrition. Choice D is incorrect. Severe burns or trauma can put a greater demand on the body for nutrients than what can safely be consumed via the GI system. Parenteral nutrition may be indicated to supplement the additional nutrients without overloading the GI system with excessive caloric volume.

The nurse cares for a client diagnosed with end-stage renal disease who just returned from initial hemodialysis. Which of the following assessment findings is of the highest concern? A. Headache and nausea [45%] B. Scant blood on the AV fistula [10%] C. Potassium 4.9 mEq/L [12%] D. Hemoglobin 8.8 mg/dL [33%]

Explanation Choice A is correct. Headache and nausea may be a manifestation associated with dialysis disequilibrium syndrome (DDS). This is a complication experienced by clients undergoing their first dialysis and may range from mild to severe. Choices B, C, and D are incorrect. Scant blood on the AV fistula is a benign finding. Bleeding is a complication following hemodialysis, but scant blood is not indicative of such a complication. Elevated potassium and low hemoglobin are expected in end-stage renal disease (ESRD) clients. Additional Info DDS is usually self-limiting and is common during the first treatment. This is caused by removing urea, which causes a fluid shift that may lead to cerebral edema. The nurse should remain with the client and institute fall precautions. Notifying the primary healthcare provider (PHCP) should be done despite most DDS being self-limiting. Other complications of hemodialysis include hypotension, bleeding, angina, and cramps.

The nurse is taking vital signs for a client who has a chest tube in place. While counting the client's respirations, the nurse notes that the water in the water-seal-chamber is fluctuating. Which action by the nurse is most appropriate based on this finding? A. Finish counting the client's respirations [82%] B. Empty the water-seal chamber [3%] C. Assist the client with incentive spirometry [6%] D. Notify the charge nurse [8%]

Explanation Choice A is correct. It is appropriate for the nurse to finish counting the client's respirations and continue to monitor them as normal. Fluctuations of water in the water-seal chamber with inspiration and expiration are a sign that the drainage system is patent. Normally, the water level will increase when the client breathes in, and then decrease when they breathe out. This is due to changes in intrathoracic pressures. Choice B is incorrect. The nurse should not empty the water-seal chamber. This would cause a break in the closed drainage system and could result in injury to the client. Choice C is incorrect. It is not necessary for the nurse to help the client with incentive spirometry (IS) based on this finding. The nurse has noted an expected finding of the chest-tube system and can continue to assess the client as normal. Choice D is incorrect. It is not necessary for the nurse to notify the charge nurse based on this finding. The nurse has noted an expected finding of the chest-tube system and can continue to monitor the client as normal. NCSBN Client Need Topic: Reduction of Risk Potential, Subtopic: Potential for Complications of Diagnostic Tests/Treatments/Procedures; Respiratory Additional Info

The nurse is caring for the body of a Muslim patient who has recently passed away. Knowing the after-death practices preferred in this religion, the nurse performs which of the following actions? A. Request all non-Muslims handling the body to wear gloves. [40%] B. Call a priest to perform the Last Rites. [11%] C. Discuss the death with the collective members of the family. [39%] D. A leaf of basil should be placed on the deceased person's tongue. [11%]

Explanation Choice A is correct. Most branches of the Islamic faith request that the deceased body be handled only by Muslims of the same gender. If no Muslim practitioners are available, then staff should wear gloves. Choice B is incorrect. Last Rites are performed on the very sick or dying members of the Catholic faith. Choice C is incorrect. Death and dying are not generally discussed with that outside of the family in the Islamic faith. Choice D is incorrect. A leaf of basil is generally placed on the deceased's tongue of those practicing Hinduism. NCSBN client need Topic: Psychosocial Integrity: Cultural awareness and end-of-life care

Religious and cultural rituals/practices often surround death. Which of the following populations prefer cremations rather than burying the remains of the deceased person? A. Hindus [54%] B. Islam [12%] C. Mormons [13%] D. Eastern orthodox [21%]

Explanation Choice A is correct. The Hindus prefer cremations rather than burying the remains of the deceased person. The ashes are then typically spread over the holy river. Cremations are viewed as discouraged or forbidden among those who practice Islam, Mormonism, and the Eastern Orthodox religion. Choice B is incorrect. Cremations are viewed as discouraged or forbidden among those who practice the Islam religion. Choice C is incorrect. Cremations are viewed as discouraged or forbidden among those who practice the Mormon faith. Choice D is incorrect. Cremations are viewed as discouraged or forbidden among those who practice the Eastern Orthodox religion.

The first bodily area to be washed with a complete bed bath is the: A. Inner canthus of the right eye [78%] B. Cheeks [2%] C. Forehead [20%] D. Chin [0%]

Explanation Choice A is correct. The first bodily area to be washed with a complete bed bath is the inner canthus of either eye, including the right or left eye. The washing is done from the inner to the outer canthus of the eye. The next steps for the bath are the rest of the face, the upper chest, the arms, and hands, after which you would proceed downward on the body from the head to the toes. Choice B is incorrect. Although the cheeks are washed near the beginning of a complete bed bath, washing either cheek is not the first bodily area to be washed. Choice C is incorrect. Although the forehead is washed near the beginning of a complete bed bath, washing the forehead is not the first bodily area to be washed. Choice D is incorrect. Although the chin is washed near the beginning of a complete bed bath, washing the chin is not the first bodily area to be washed.

The critical care nurse is caring for a client receiving hemodynamic monitoring. After reviewing the client's central venous pressure, the nurse should take which action based on the client's 0100 CVP reading? See the image below. A. Assess the client for fluid volume overload [73%] B. Obtain a prescription for saline fluid bolus [4%] C. Document the findings and continue to monitor [16%] D. Place the client in a side-lying position. [8%]

Explanation Choice A is correct. The normal central venous pressure (CVP) is 2 to 8 mmHg. CVP measures the amount of fluid that returns to the right atrium (or preload). This client's CVP has been trending upward, and the nurse should assess the client for hypervolemia, specifically, manifestations of right-sided heart failure. Choices B, C, and D are incorrect. Infusing additional volume may be detrimental. This would be an appropriate intervention for a client with a low CVP. These values are abnormal, and the nurse should not document and continue to monitor. This value requires action on the part of the nurse. The side-lying position is unnecessary for this client, and if the assessment confirms fluid volume overload, the client should be positioned as high-Fowlers. Additional Info CVP measurement can be obtained through a central venous catheter. The normal value is 2 to 8 mmHg. A low CVP may be caused by venous dilation and low fluid volume. A high CVP may be caused by hypervolemia, tension pneumothorax, or right-sided heart failure.

The nurse is caring for a client diagnosed with multiple myeloma. The nurse reviews the client's lab values and notes a serum calcium level of 14 mg/dL. What is the priority action the nurse should take? A. Notify the physician [86%] B. Document the finding [5%] C. Continue to monitor the patient [8%] D. Remove the patient from the telemetry monitor [1%]

Explanation Choice A is correct. The normal range for serum calcium is 8.4-10.2 mg/dL. This client's serum calcium level is above 10.2 mg/dL; therefore, the client is experiencing hypercalcemia. At a calcium level of 14 mg/dL, most clients may experience symptoms. Often, these may include polyuria, polydipsia, and dehydration. If not addressed, clients may develop renal failure and altered mental status. The nurse must notify the physician regarding this abnormal lab value. Choices B and C are incorrect. It is inappropriate for the nurse to document the finding or just continue to monitor. The nurse has correctly identified that this lab value is out of the normal range and must report the finding to the attending physician. Choice D is incorrect. It is inappropriate for the nurse to remove the client from the telemetry monitor. Not only has the nurse identified that this finding falls outside of normal limits and needs to notify the attending physician, but the nurse should also be aware that a client experiencing hypercalcemia may have EKG changes such as a shortened QT interval and a prolonged PR interval. Cardiac monitoring is essential for this client. NCSBN Client Need Topic: Physiological Adaptation, Subtopic: Fluid & Electrolytes Imbalances

The nurse is preparing to give an intramuscular (IM) injection into the client's vastus lateralis. It would indicate the correct technique if the nurse A. palpates to find greater trochanter and knee joints; divide the vertical distance between these two landmarks into thirds; inject into the middle third. [34%] B. locates acromion process; inject only into the upper third of muscle that begins about two fingerbreadths below the acromion. [10%] C. locates the greater trochanter, iliac tubercle, and iliac crest; places palm over the greater trochanter, over iliac tubercle, along the ileum; inject into center of V formed by the fingers. [42%] D. displaces the skin by pulling the skin down or to one side about 1 inch with the nondominant hand before administering the injection. [13%]

Explanation Choice A is correct. This is the appropriate anatomical landmark for giving an IM in the vastus lateralis. To locate the vastus lateralis, palpate to find greater trochanter and knee joints; divide vertical distance between these two landmarks into thirds; inject into middle third. Choices B, C, and D are incorrect. Locating the acromion process is a landmark involved with giving an IM in the deltoid muscle. Locating greater trochanter, iliac tubercle, and iliac crest is involved in giving an IM ventrogluteal. Displacing the skin by pulling the skin down or to one side about 1 inch is relevant to the Z-track technique which is commonly used when giving an IM injection. Additional Info A key advantage of using the vastus lateralis is that an intramuscular (IM) injection may be given if the client is supine, side-lying, or sitting. Aspiration for routine injections into deltoid or vastus lateralis is not indicated because there are no large blood vessels in these locations.

You are walking through the cafeteria in your medical center. As you enter you see a female patient on the floor in a puddle of soda. Which of the following should you document on the accident/incident report? The client: A. Was found on the floor. [93%] B. Slipped on soda and fell. [6%] C. Slipped and hit their head. [1%] D. Was very careless. [0%]

Explanation Choice A is correct. You would document that the female patient was found on the floor. Only objective facts are documented on incident or accident reports. You cannot state that the client slipped on soda because this is not a fact that you can substantiate; it is your opinion about how this accident happened. You would also not state that the client slipped and hit their head because you did not witness this. And, finally, you would not document that the client was careless; that is simply an assumption and it is not an objective fact. Choice B is incorrect. You cannot state that the client slipped on soda because this is not a fact that you can substantiate; it is your opinion about how this accident happened. Choice C is incorrect. You would not state that the client slipped and hit their head because you did not witness this. Choice D is incorrect. You would not document that the client was careless; that is simply an assumption and it is not an objective fact.

The nurse is discussing ocular disorders with a group of nursing students. Which of the following statements would be correct for the nurse to make? Select all that apply. A. Cataracts are caused by increased ocular pressure (IOP). [9%] B. Graves' disease may cause exophthalmos. [33%] C. Macular degeneration is manifested by loss of peripheral vision. [19%] D. Angle-closure glaucoma is manifested by headache and eye pain. [26%] E. Hyphema results in increased aqueous humor in the anterior chamber. [13%]

Explanation Choice B and D are correct. Graves' disease may cause a client to develop exophthalmos. Angle-closure glaucoma is a medical emergency where the IOP is greater than 30 mmHg, and the client has manifestations such as eye pain, headache, blurred vision, and reddened eye appearance. Choices A, C, and E are incorrect. Increased IOP is a central feature of glaucoma. Cataracts is a disorder of the lens as it causes the client to have difficulty discriminating colors and seeing in low light. Opacities can commonly be seen in the affected eye. Macular degeneration causes central vision loss, not vision loss in the peripheral fields. A hyphema is caused by blood in the eye's anterior chamber. NCLEX Category: Physiological Adaptation Activity Statement: Illness management Question type: Analysis Additional Info In managing a hyphema and angle-closure glaucoma, the nurse should be aware of the following: A hyphema is an ocular emergency that has been caused by blood in the anterior chamber. This injury results from trauma and should be addressed promptly with interventions such as elevating the head of the bed to 30 degrees and shielding the affected eye. Angle-closure glaucoma is an ocular emergency that requires the client to receive prescribed agents such as timolol to lower intraocular pressure. The client should be placed supine, which will assist in the lens falling away from the iris, decreasing the pupillary block.

Select an appropriate nursing diagnosis for your client who is affected with hyperalgesia. A. At risk for inadvertent narcotic overdoses related to hyperalgesia. [43%] B. At risk for abnormal and irreversible pain related to hyperalgesia. [24%] C. At risk for somatic pain related to hyperalgesia. [21%] D. At risk for visceral pain related to hyperalgesia. [12%]

Explanation Choice B is correct. "At risk for abnormal and irreversible pain related to hyperalgesia" is an appropriate nursing diagnosis for a client who is affected with hyperalgesia. Hyperalgesia, which is synonymous with hyperpathia, is abnormal pain processing that can lead to the appearance of neuropathic pain that is irreversible if left untreated. Choice A is incorrect. "At risk for inadvertent narcotic overdoses related to hyperalgesia" is not an appropriate nursing diagnosis for a client who is affected with hyperalgesia. Hyperalgesia is abnormal pain processing that is not associated with inadvertent narcotic overdosages. Choice C is incorrect. "At risk for somatic pain related to hyperalgesia" is not an appropriate nursing diagnosis for a client who is affected with hyperalgesia. Hyperalgesia can lead to neuropathic pain, but not somatic nociceptive pain. Choice D is incorrect. "At risk for visceral pain related to hyperalgesia" is not an appropriate nursing diagnosis for a client who is affected with hyperalgesia. Hyperalgesia can lead to neuropathic pain, but not visceral nociceptive pain.

Your client is on complete bed rest for 7 days. Which of the following is the highest priority nursing diagnosis for this client? A. At risk for severe sensory deprivation related to complete bed rest. [5%] B. At risk for venous stasis related to complete bed rest. [74%] C. At risk for decreased muscular strength related to complete bed rest. [18%] D. At risk for urinary stasis related to complete bed rest. [3%]

Explanation Choice B is correct. "At risk for venous stasis related to complete bed rest" is the highest priority nursing diagnosis for a client who is on complete bed rest for seven days. Venous stasis adversely affects the circulatory system, and this venous stasis can lead to life-threatening complications such as venous stasis and pulmonary emboli. According to the "ABCs" of the airway, breathing, and cardiovascular status, Maslow's Hierarchy of Needs, and the MAA-U-AR method of priority setting method, all establish the highest priorities as A: Airway, B: Breathing, and C: Circulation in that decreasing order of preference. Choice A is incorrect. "At risk for severe sensory deprivation related to complete bed rest" is not the highest priority nursing diagnosis for a client who is on complete bed rest for seven days. Choice C is incorrect. "At risk for decreased muscular strength related to complete bed rest" is not the highest priority nursing diagnosis for a client who is on complete bed rest for seven days. Choice D is incorrect. "At risk for urinary stasis related to complete bed rest" is not the highest priority nursing diagnosis for a client who is on complete bed rest for seven days.

The nurse is caring for a client experiencing an adrenal crisis (Addisonian crisis). The nurse should be prepared to administer which intravenous fluid? A. Lactated Ringers (LR) [13%] B. 0.9% saline [48%] C. Dextrose 5% in water (D5W) [18%] D. Dextrose 5% in water and Lactated Ringers (D5LR) [20%]

Explanation Choice B is correct. A client experiencing an adrenal crisis (Addisonian crisis) tends to have significant hypovolemia and hyponatremia. Because of the deficiency of steroid hormones, distributive shock may follow. Restoring the circulatory volume is essential in the management of this crisis. Isotonic solutions such as 0.9% saline or D5NS ( dextrose 5% in water combined with 0.9% saline) must be used. Isotonic saline can address both hypovolemia and hyponatremia in the adrenal crisis. If there is concomitant hypoglycemia, the D5NS solution is preferred to increase the glucose, sodium, and circulatory volume. Choices A, C, and D are incorrect. Although lactated ringers (LR) is an isotonic solution, it is inappropriate in managing an adrenal crisis because the client is experiencing concomitant hyponatremia. LR will not correct the hyponatremia ( Choice A). D5W is hypotonic and would be detrimental if given by itself because it would increase the free water and lower the sodium further by dilution ( Choice C). D5LR has a limited benefit in an adrenal crisis because of its inability to raise sodium levels ( Choice D). Learning Objective Understand that the Addisonian crisis can result in a distributive shock and hyponatremia. Isotonic ( 0.9%) saline or D5NS are preferred fluids in managing the Addisonian crises. Additional Info Addison's disease is an autoimmune condition in which the client has insufficient cortisol and aldosterone. The mainstay treatment is lifelong corticosteroid replacement with hydrocortisone. The dosage of replacement hormones may need to be increased if the client experiences increased demands from stressful events or illnesses. Failure to increase the replacement doses to meet the demand will result in adrenal crisis and shock. During an adrenal crisis, the priority treatment is administering hydrocortisone intravenously. The client is often volume depleted, hypoglycemic, and hyponatremic and will need rapid fluid resuscitation. Dangerously high potassium levels ( hyperkalemia) are also evident in an adrenal crisis and require cardiac monitoring and potassium-reducing medications such as sodium polystyrene.

The nurse preceptor is observing a newly hired nurse care for assigned clients. It would require follow-up by the nurse preceptor if the newly hired nurse is observed doing which of the following? A. Humidifies nasal cannula oxygen for a client with sarcoidosis. [11%] B. Secures a suprapubic catheter tubing to a client's inner thigh. [20%] C. Places a client with varicella-zoster in airborne and contact isolation. [8%] D. Suctions a tracheostomy for 15 seconds as they remove the catheter. [60%]

Explanation Choice B is correct. A suprapubic catheter should be looped and taped to the client's abdomen. Taping it to the abdomen decreases the tension on the tubing, decreasing its risk of dislodging. Choices A, C, and D are incorrect. Humidifying nasal cannula oxygen is appropriate, especially when used for the long term or at least 4 liters per minute. Varicella-zoster warrants airborne and contact precautions. Suctioning a tracheostomy for 10-15 seconds is appropriate if the catheter is removed in a twirling motion. Additional Info A suprapubic catheter is inserted percutaneously above the symphysis pubis into the bladder. The indications for the catheter include urethral strictures or trauma. Nursing care involves preventing infection by instructing the client to perform hand hygiene and clean the site with warm soapy water, then applying a sterile dressing around the tube.

A 72-year-old elderly patient is brought to the emergency department from a local nursing home. The CNA that arrived with the patient states that she started feeling weak earlier today and has been slightly more confused than usual. Her vital signs are shown in the exhibit. What is the nurse most concerned about? See the exhibit. A. Transient ischemic attack (TIA) [16%] B. Sepsis [67%] C. Cerebrovascular accident (CVA) [11%] D. Dementia [6%]

Explanation Choice B is correct. According to this patient's vital signs, she is developing early signs of sepsis. Sepsis can be identified by an elevated temperature (above 100.4 degrees Fahrenheit, or 38 degrees Celcius), heart rate, respiration, and low blood pressure. Her heart rate is still within normal limits, however, this patient does have increased respiration, lower blood pressure, and a low-grade fever. In elderly patients, these vital signs need to be identified as soon as possible to prevent complications and ensure timely interventions. Choice A is incorrect. There is no data to prove that this patient is suffering from a TIA. Choice C is incorrect. There is no data to prove that this patient is suffering from a stroke. Choice D is incorrect. This patient is elderly and may already have dementia, but there is no data to prove that this patient has dementia. NCSBN Client Needs Topic: Reduction of Risk Potential, Sub-Topic: Changes/Abnormalities in Vital Signs, Infection

A nurse is caring for a 13-year-old boy is scheduled to have a surgical repair of a spinal curvature. The adolescent will be hospitalized for approximately 2 weeks. While planning care, which nursing intervention will be most helpful during the hospital stay? A. Instruct parents to room-in with him. [5%] B. Encourage the patient to bring homework assignments to the hospital. [38%] C. Have the client complete a puzzle. [45%] D. Encourage the patient to go to the activity room daily. [12%]

Explanation Choice B is correct. Adolescents need to establish their identity, which includes developing a mature sense of responsibility/independence. Providing the patient with his schoolwork will keep him connected to his peer group and give him a sense of accomplishment. Choice A is incorrect. Unlike infants and toddlers, adolescents are less likely to experience separation anxiety. Choice C is incorrect. Completing a puzzle is a task more appropriate for a school age child in Erikson's stage of industry vs. inferiority. In this stage, school age children seek to develop a sense of industry, compare themselves with their peers, and see how they measure up. The adolescent client in this question is in Erikson's stage of identity vs. role confusion. Helping the client complete his homework and promoting responsibility and independence is more appropriate for this developmental stage. Choice D is incorrect. Preschool and school-aged children may benefit from the social interaction that is offered in the activity room. Adolescents, however, are more likely to enjoy independent activities, especially after a surgical procedure. NCSBN: Client NeedTopic: Health Promotion and Maintenance, Sub-Topic: Developmental stages and Transitions.

The clinic nurse notices bruises at multiple stages of healing on a 2-year-old. The nurse also sees a couple of burns on the toddler's trunk. What would be the nurse's most appropriate action? A. Confront the child's parents [3%] B. Call Child Protective services [90%] C. Check the child again after two weeks [0%] D. Call the physician [6%]

Explanation Choice B is correct. Bruises and burns in a child indicate abuse. Once the nurse suspects child abuse, he/she is responsible for notifying Child Protective Services. Choice A is incorrect. The nurse is not in a position to confront the parents regarding the bruises and burns of the child, especially if the nurse is in a home visit. This would compromise the nurse's safety. Choice C is incorrect. The nurse should not wait for another moment to report the child as additional harm might come to the child. Choice D is incorrect. Calling the physician does not remove the child from danger. The nurse may need to call the physician, but the nurse should call child protective services first.

Pain management for burns primarily consists of morphine delivered intravenously. The reason for this is: A. The IV route delays absorption to provide continuous pain relief. [11%] B. The IV route facilitates absorption as muscle absorption is not dependable. [65%] C. The IV route allows for discontinuance in the event of respiratory depression. [12%] D. The IV route prevents further pain from IM injections. [12%]

Explanation Choice B is correct. In clients with burns, they are usually hemodynamically unstable, and tissue perfusion is compromised. Medications remain in the subcutaneous tissue with the fluid in the interstitial spaces in the acute phase of the injury. Because of this, the IM route is avoided until stability has been achieved. Choices A, C, and D are incorrect. The I.V. route of administering medications facilitates increased absorption of drugs, not decreased intake. Choice A is incorrect. In the event of respiratory depression, the IV route would hasten the process, making choice C incorrect. The goal in IV administration is not for the primary reason of avoiding additional client pain. Choice D is also wrong.

Which of the following statements indicates body image distortion in a patient with anorexia nervosa? A. "I wish I looked like my mom." [1%] B. "I hate how my body looks." [21%] C. "I wish I could wear tank tops." [2%] D. "I'm so overweight." [75%]

Explanation Choice D is correct. Patients with anorexia perceive themselves to look differently than they do. Many of these patients see someone in the mirror who weighs more than their desired weight. Despite being too thin, this client will not eat in hopes of getting the perfect body. Choices A, B, and C are incorrect. Although these are signs of altered body image, these statements do not reflect body image distortion. NCSBN Client Need Topic: Health Promotion and Maintenance, Subtopic: Factors Affecting Self-Perception

Your pregnant client has been hospitalized with hyperemesis gravidarum. She is given ondansetron to treat this illness. What serious side effects should the hospital nurses be watching for? A. Continued nausea and vomiting [10%] B. Prolonged QT interval [43%] C. Respiratory distress [29%] D. Constipation [17%]

Explanation Choice B is correct. Prolonged QT intervals have been noted as a severe side effect of ondansetron. This medication is used to treat hyperemesis gravidarum when the patient is losing weight and or unable to cope with pregnancy-related nausea. Choice A is incorrect. While the nurse should monitor for the resolution of nausea and vomiting, the continuation of emesis is not a side effect of ondansetron. Choice C is incorrect. Respiratory distress is not generally associated with ondansetron. Choice D is incorrect. Constipation is not a severe side effect of ondansetron. NCSBN client need Topic: Physiological Integrity: Pharmacological and Parenteral Therapies

When teaching parents about the pros and cons of their children sleeping with them, which of the following information should the nurse give the parents? A. If you give your child more attention during the day they will not want to sleep with you at night. [10%] B. Sleeping with parents can contribute to Sudden Infant Death Syndrome (SIDS). [86%] C. Children should never be allowed to sleep with their parents. [2%] D. You could be accused of sexual abuse if you allow your child to sleep with you. [2%]

Explanation Choice B is correct. Research has shown that kids sleeping with adults can contribute to sudden infant death syndrome (SIDS). The mechanism is believed to be the rebreathing of carbon dioxide as the sleeping child snuggles against the parent. Babies should stay in their parents' room at night for a full year, according to recommendations released by the American Academy of Pediatrics. Babies shouldn't share a bed with parents due to the increased risk of sudden infant death syndrome (SIDS). The safest spot for infant sleep is on a firm surface such as a crib or bassinet without any soft bedding, bumpers, or pillows. Choice A is incorrect. Increased attention during the day will not stop a child from wanting to sleep with their parents. Choice C is incorrect. In some cultures, it is the norm for children to sleep with their parents. Some occasions provide reasons a parent might want the child to sleep with them, such as illness, nightmares, or bedroom space. Unless parents want this to become a habit, they will have to take measures to be sure it is only for the duration of the circumstance. Choice D is incorrect. This is incorrect. The most appropriate answer to this question is choice B. NCSBN Client Need Topic: Health Promotion and Maintenance, Subtopic: Discharge Teaching for Care of the Newborn

While caring for a client who requires a mechanical ventilator for breathing, the high-pressure alarm goes off on the ventilator. What is the first action the nurse should perform? A. Disconnect the client from the ventilator and use a manual resuscitation bag. [10%] B. Perform a quick assessment of the client's condition. [84%] C. Call the respiratory therapist for help. [1%] D. Press the alarm reset button on the ventilator.

Explanation Choice B is correct. Several situations can cause the high-pressure alarm to sound. An assessment of the client will tell the nurse whether the alert was triggered by something simple, such as the patient coughing, or by a more difficult situation that might require using a manual resuscitation bag and calling the respiratory therapist. Several things can trigger pressure alarms on mechanical ventilators. Some of the most common causes of high-pressure alarm triggers include water in the ventilator circuit, increased or thicker mucus or other secretions blocking the airway (caused by not enough humidity), bronchospasm, coughing, gagging, or "fighting" the ventilator breath. Regardless of the cause of the triggered alarm, the priority for nurses is to evaluate the patient's status first. Choice A is incorrect. If the patient is struggling for air, the nurse should disconnect the ventilator and use a manual resuscitation bag. This will be evident when the patient is assessed, which is the first nursing action that should be taken. Choice C is incorrect. Although the respiratory therapist may need to be called, this should not be the nurse's first response. Choice D is incorrect. The reset button may need to be engaged. However, the patient's status should be the nurse's priority. NCSBN Client Need Topic: Physiological Integrity, Subtopic: Reduction of Risk Potential

The nurse evaluating an oncology patient's chart notes that the patient has a tumor in his lung measuring about 4.3 cm in size and accompanying pneumonitis. His cancer does not invade the entire lung and has no metastasis or lymph node involvement. Using the TNM staging system, how would the nurse best classify this patient's tumor? A. T3 N3 M1 [4%] B. T2 N0 M0 [62%] C. T1 N1 M0 [25%] D. T2 N1 M0 [10%]

Explanation Choice B is correct. The TNM tumor staging system explores tumor size (T), node involvement (N), and distant metastasis (M). This patient has a small tumor measuring 4.3 cm limited to one portion of the lung, giving it a T staging of T2. Without nodal involvement or metastasis, both N and M are 0. This question is intended to test the representation of N0 and M0 for negative lymph nodes and negative metastases, respectively. The nurse is not required to know "T" staging details. Choice A is incorrect. T3 N3 M1 defines a tumor higher than 7 cm, metastasis in all contralateral lymph nodes, and distant metastasis. Choice C is incorrect. T1 N1 M0 designates a tumor that is self-contained and less than 3 cm in size, has some lymph node involvement, and no metastasis. Choice D is incorrect. T2 N1 M0 implies a tumor that is between 3 and 7 cm in size, has lymph node involvement without distant metastasis. NCSBN client need Topic: Physiological Adaptation, Illness Management

The nurse in the delivery room has just assisted in the delivery of a newborn and is now attempting to deliver the placenta. The nurse understands that expulsion of the placenta would trigger all of the following processes, except: A. Decrease in progesterone [15%] B. Decrease in estrogen [17%] C. Increase in prolactin [21%] D. Production of oxytocin [46%]

Explanation Choice D is correct. This is an incorrect statement, therefore the correct answer to the question. Oxytocin production is stimulated by suckling. Suckling stimulates the posterior pituitary gland to produce oxytocin, causing the release of milk from alveoli into the ducts. Choices A, B, and C are incorrect. This is a correct statement. When the placenta is expelled, this causes a decrease in estrogen and progesterone that causes the anterior pituitary gland to increase prolactin.

The client is upset because they just found out that they have syphilis. They tell the nurse, "This is so upsetting! Does everyone need to know?" Which of the following responses, if made by the nurse, is the most therapeutic? A. "We need to report this case to the Public Health Department and they will call your past partners." [25%] B. "According to the Health Insurance Portability and Accountability Act (HIPAA), I can't tell anyone without your permission." [13%] C. "You really should contact your sexual partners, so they can be treated too." [12%] D. "I understand you're upset. I'll stay here with you so that you can talk about it." [49%]

Explanation Choice D is correct. This response encourages verbalization of client's feelings and is an example of therapeutic communication. This communication technique focuses on prioritizing the client's physical, mental and emotional well being. Choice A is incorrect. While it is true that the Public Health Department will attempt to notify any sexual partners of the client reports, This is not the most therapeutic response the nurse should make to her client. Immediately after learning about the diagnosis, the client will need the nurse to be supportive and allow them to express their emotions. Simply telling the client that the Public Health Department will attempt to notify their sexual partners is not therapeutic for the client at this time; even though it is indeed the role of the Public Health Department is to focus on disease prevention and treatment so it does not become a larger problem in the community. Choice B is incorrect. This statement is incorrect as the disease needs to be reported to the health department. The public health department will get the minimal amount of information necessary. This is not a violation of HIPAA. Choice C is incorrect. Informing prior partners that they may have been exposed to syphilis is an appropriate action the client may choose to take. However, at this time, it is not the priority statement the nurse should make. At this time, the nurse should focus on proving therapeutic responses to help her patient cope and process this stressful event. Learning Objective Learning Objective: Apply the use of therapeutic communication to promote and support the coping abilities of the client Additional Info Syphilis is a bacterial infection usually spread by sexual contact that starts as a painless sore (chancre). This occurs around 21 days after exposure to the bacteria that causes syphilis. Syphilis develops in stages, and symptoms vary with each stage. The first stage involves a painless sore on the genitals, rectum, or mouth. After the initial sore heals, the second stage is characterized by a rash. Then, there are no symptoms until the final stage which may occur years later. This final stage can result in damage to the brain, nerves, eyes, or heart. Syphilis is treated with penicillin. Sexual partners should also be treated. All 50 states in the U.S. require that syphilis cases be reported to the state or local public health agency so that exposed individuals may receive treatment.

A parent is sobbing loudly at the bedside of their infant, who was just born with an omphalocele. She wails, "I cannot believe my baby was born this way! He will surely die!" Which of the following statements by the nurse is most appropriate? A. "Oh, hush, this is not so bad." [0%] B. "I am so sorry. This is certainly serious, but there is a treatment that can repair the omphalocele and we will take excellent care of your baby." [65%] C. "Your baby will not die; this can be fixed!" [2%] D. "I understand you must be upset. Would you like to hold your baby while he is still here?" [33%]

Explanation Choice B is correct. This statement is a fitting example of therapeutic communication. It validates the fear and emotions the parent is experiencing and provides education about the infant's treatment and prognosis. This opens up the conversation for dialogue and further information about what the parent can expect. Choice A is incorrect. This is not a therapeutic statement. It would be inappropriate for the nurse to say this to a distressed parent. Choice C is incorrect. It is inappropriate for the nurse to make statements such as, "Your baby will not die!" since it may sound like a promise to the parent. It is impossible to know what will happen with this patient, so if they did pass away during surgery, the parent might be very angry with the nurse. As a general rule, never promise a specific outcome to a patient or parent. Choice D is incorrect. While this statement sounds therapeutic, it does not provide any education and validates the parent's misunderstanding about their infant dying. An omphalocele repair is very successful, with over 90% of infants surviving. The nurse should use therapeutic communication to validate the parent's emotions while providing education about realistic expectations to understand the prognosis better. NCSBN Client Need Topic: Psychosocial Integrity, Subtopic: Pediatrics - Gastrointestinal

The nurse is caring for a client with the following tracing on the electrocardiogram (ECG). The nurse identifies this tracing as See the image below. A. sinus tachycardia. [8%] B. supraventicular tachycardia. [70%] C. ventricular tachycardia. [12%] D. atrial flutter. [9%]

Explanation Choice B is correct. This tracing reflects SVT, which is concerning because of the very high rate. The rate may be as high as 100 to 280 beats/min in adults. Characteristically, SVT does not have P-waves as they are buried in the T-waves. Choices A, C, and D are incorrect. Sinus tachycardia (ST) is excluded because ST has noticeable P-waves proceeding each QRS complex. Ventricular tachycardia (VT) is excluded because while this dysrhythmia may have a rate up to 180 beats/min, the QRS complex is noticeably widened. Atrial flutter is excluded because of its gross irregularity. Additional Info During SVT, P waves may not be visible, because the P waves are embedded in the preceding T wave. A client with SVT may be asymptomatic. If the client is symptomatic, they may exhibit manifestations such as palpitations, dizziness, dyspnea, and nervousness. Treatment includes vagal maneuvers. Vagal maneuvers include having the beardown, blowing through a straw, having the primary healthcare provider (PHCP) perform a carotid massage, and, if the client is an infant, applying an ice pack firmly to the infant's head. If these measures are ineffective, the nurse should prepare to administer the prescribed adenosine by rapid intravenous push (IVP) that is followed by a flush of 0.9% saline. When adenosine is administered, the emergency (code) cart should be nearby, and the nurse should always have additional personnel in the room.

While admitting a patient, the nurse begins to review information regarding advanced directives. Still, the patient becomes agitated and refuses to discuss the issue or accept a handout about the topic. Which is the appropriate nursing action? A. Leave the handout on the patient's bedside table instructing him that he must review the content. [3%] B. Document the patient's refusal, using the patient's own words, in quotes. [47%] C. Explain to the patient that he must make decisions about accepting or refusing treatment while in the hospital. [15%] D. Request an assessment of the patient's competency related to making decisions about advanced directives.

Explanation Choice B is correct. While the Patient Self-Determination Act requires health care facilities to provide information about the patient's right to refuse or accept treatment, the patient has the right to withdraw that information. Should the patient decline verbal and written information about advanced directives, the nurse should document that information was offered, and document the patient's refusal, quoting the patient's statements. Choices A and C are incorrect - The patient has the right to autonomy and self-determination, including refusing information regarding advanced directives. He is not required to have advanced instruction in place while in the hospital. Choice D is incorrect - The patient's refusal to accept information about advanced directives is not an indication of the patient's level of competence. Bloom's Taxonomy - Analyzing

The nurse is developing a health promotion teaching plan for a community group of middle-aged adults. Information about which immunizations should be included? A. Pneumococcal and meningococcal [9%] B. Pertussis, influenza, and meningococcal [18%] C. Influenza and pneumococcal [72%] D. Meningococcal and pertussis [1%]

Explanation Choice C is correct. Adults aged 50 years or older tend to have multiple chronic illnesses, in addition to an aging immune system. For this reason, the influenza vaccine is highly recommended. Choice A is incorrect. The pneumococcal vaccine protects against the most common pathogens that cause pneumonia. It is given every ten years. Choice B is incorrect. Immunization for pertussis is only indicated in children. Choice D is incorrect. Meningococcal vaccine is appropriate for adolescents and young adults living in congregate housing. NCSBN Client Need Topic: Physiological Integrity, Subtopic: Physiological Adaptation

When communicating with a client who speaks a different language the nurse should do which of the following? A. Speak loudly and slowly [1%] B. Stand close to the client and speak in an exaggerated volume [0%] C. Arrange for an interpreter when communicating with the client [96%] D. Speak to the client and family together to promote comprehension [3%]

Explanation Choice C is correct. Arranging for an interpreter would be the best thing to do when communicating with a client who speaks a different language. Choices A and B are incorrect. These are inappropriate and ineffective ways of communicating. Choice D is incorrect. This would be inadequate because it does not ensure correct translation and it violates the patient's right to privacy.

The 8 to 10-week ultrasound verifies all of the following, except: A. Estimated Due Date (EDD) [10%] B. Pelvic shape [8%] C. The absence of fetal abnormalities [70%] D. Confirm pelvic health assessed at the first prenatal appointment [12%]

Explanation Choice C is correct. At the 8 to 10-week ultrasound; the radiologist will not be able to rule out all fetal abnormalities. Choices A, B, and D are incorrect. Ultrasounds performed during this period do help determine the due date as well as verify the pelvic shape and health. NCSBN Client need Topic: Maintenance and Health Promotion, Ante / Intra / Postpartum Care

The nurse is setting up the room for a patient newly diagnosed with Celiac disease. She knows to place the patient on which of the following precautions? A. Droplet precautions [1%] B. Contact precautions [8%] C. Standard precautions [80%] D. Neutropenic precautions [10%]

Explanation Choice C is correct. Celiac disease requires standard precautions. It is not an infectious disease and is not transmitted from person to person; therefore, there is no reason to initiate any additional precautions. Choice A is incorrect. Droplet precautions are not indicated for the patient with Celiac disease. It is not an infectious disease and is not transmitted from person to person. Droplet precautions are indicated when there is an infection that can spread by speaking, sneezing, or coughing near someone. Examples include influenza, adenovirus, and rhinovirus. Choice B is incorrect. Contact precautions are not indicated for the patient with Celiac disease. It is not an infectious disease and is not transmitted from person to person. Contact precautions are indicated when there is an infection that can spread by touching the patient or items in the room. Examples include MRSA and VRE. Choice D is incorrect. Neutropenic precautions are not indicated for the patient with Celiac disease. Neutropenic precautions are indicated when the patient is immunocompromised and at additional risk for infection. Examples of patients on neutropenic precautions may be those who have received a transplant, or who have a low absolute neutrophil count due to cancer or chemotherapy. NCSBN Client Need: Topic: Effective, safe care environment, Subtopic: Infection control and safety; Pediatrics - Gastrointestinal

When teaching medication safety to the parent of a toddler, which statement by the parent would be a cause for concern? A. "I always check to make sure the safety cap clicks when I close it." [2%] B. "We store all of our medicines on a really high shelf." [4%] C. "To get her to take her medicine, we tell her it's candy." [76%] D. "We store our medicines and vitamins together."

Explanation Choice C is correct. Children should never be told that medication is candy. Choice A is incorrect. It is an excellent practice to make sure safety caps are adequately screwed on; therefore, this option is not a cause for concern. Choice B is incorrect. This answer does not reflect a reason for concern. Choice D is incorrect. There is nothing wrong with storing vitamins and medicines together as long as they are both kept out of the reach of children. NCSBN Client Need Topic: Safe and Effective Care Environment, Subtopic: Safety and Infection Control; Pediatric Health Promotion and Safety

Following treatment for a fracture, a client is now undergoing rehabilitation. His regimen involves performing isometric exercises. Which action is evidence that the client has fully understood the proper technique? A. The client exercises both extremities simultaneously [29%] B. The client knows that his heart rate should be monitored while exercising [10%] C. The client practices forced resistance against stable objects [45%] D. The client swings his limbs through their full range of motion [17%]

Explanation Choice C is correct. Isometric exercises involve applying pressure against a stable object, like pressing the hands together or pushing an extremity against a wall. Choices A, B, and D are incorrect. Isometric exercises do not include the simultaneous use of the extremities and neither does the swinging of limbs. Heart rate monitoring is done with aerobic exercises.

The nurse is assessing a client with a chest tube for a pneumothorax. The nurse assesses a crackling sensation beneath the fingertips around the chest tube insertion site. The nurse should take which action? A. Document the finding as normal [25%] B. Clamp the chest tube [9%] C. Notify the primary healthcare provider (PHCP) [63%] D. Apply nasal cannula oxygen [3%]

Explanation Choice C is correct. Notifying the PHCP is essential because this assessment indicates crepitus which is air trapped in and under the skin, known as subcutaneous emphysema. The PHCP needs to be notified because this is a complication, and measures such as increasing the suction on the chest tube need to be considered. Choices A, B, and D are incorrect. These actions are not appropriate. Documenting the finding as normal is inappropriate because this is a complication of chest tube therapy. Chest tubes should rarely be clamped; if they are clamped, they should be for a very brief period. This is not an oxygen issue; thus, applying nasal cannula oxygen is inappropriate and unnecessary. Additional Info Subcutaneous emphysema is the presence of air in the skin's tissue layers and is usually seen as skin swelling around the puncture site. Air in these tissues makes a crackling sound when pressure is applied to it. The presence of subcutaneous emphysema may indicate a persistent air leak caused by a puncture that tears the pleura.

The nurse is assigned to a client that just underwent a bronchoscopy to acquire a biopsy sample for his lungs. Two hours into the shift, the nurse notices the client having labored breathing. She auscultates the client's back and notes asymmetrical breath sounds. What should the nurse do first? A. Document the noted changes on the patient. [3%] B. Reassure the patient that this is a normal side effect of the procedure. [4%] C. Notify the physician. [79%] D. Prepare for intubation. [13%]

Explanation Choice C is correct. The nurse should inform the physician immediately of the impending respiratory distress brought about by the pneumothorax. Appropriate interventions include a quick insertion of a chest tube to treat the pneumothorax. Choice A is incorrect. The patient is manifesting signs of pneumothorax. The nurse can document the assessment findings she noted on the patient but not after implementing interventions to address the patient's condition. Choice B is incorrect. A pneumothorax is not a standard or common side effect of a bronchoscopy. It is a severe complication of the procedure and should be addressed accordingly. The nurse may reassure the patient to relieve anxiety, but she should initiate appropriate interventions first. Choice D is incorrect. The client is still conscious and responsive and always has respiratory functions. Interventions for a pneumothorax include the insertion of a chest tube and not endotracheal intubation.

Which action by the LPN is an example of of assault that requires immediate intervention by the nurse? A. The LPN forcibly puts the client in the "quiet" room. [22%] B. The LPN restricts the client from leaving the day room unless they put on socks. [3%] C. The LPN accompanies the client to another building. [0%] D. The LPN threatens to pull the client out of bed who is refusing to get out of bed. [74%]

Explanation Choice D is correct. Threatening a client can result in client fear, which is an example of assault. Choice A is incorrect. Forcing the client into the "quiet" room constitutes battery, which is touching the client without consent. Choice B is incorrect. Restricting the client from leaving the day room is false imprisonment. Choice C is incorrect. This does not constitute an illegal action by the LPN.

Select the hazard of immobility and complete bed rest that is accurately paired with one of its preventive measures. A. The accumulation of respiratory secretions: Oxygen supplementation therapy [5%] B. Dorsiflexion of the foot: Using a foot board or boots to maintain proper positioning [24%] C. Venous stasis: The use of a sequential compression device [65%] D. Skin breakdown: The use of a tilt table for clients at risk [6%]

Explanation Choice C is correct. Venous stasis, a complication of immobilization and bed rest can be prevented with the use of a sequential compression device (SCD), anti-embolic stockings, client positioning, range of motion exercises, and active leg exercises in bed to promote venous return and to prevent venous stasis, deep vein thrombosis, and pulmonary emboli. Choice A is incorrect. Although the accumulation of respiratory secretions is a hazard of immobility, it is not prevented with oxygen supplementation therapy. It can be prevented with adequate fluid intake, coughing, and deep breathing exercises. Choice B is incorrect. Dorsiflexion of the foot is not a hazard of immobility and bed rest (however, plantarflexion and foot drop are hazards). Plantar flexion can be prevented with a footboard or boots to maintain proper positioning and exercising the feet. Choice D is incorrect. Although skin breakdown and pressure ulcers are hazards of immobility, they are not prevented with a tilt table. Skin breakdown and pressure ulcers are prevented with good skincare, turning and positioning, and other preventive measures such as maintaining proper nutrition and avoiding the forces of pressure, friction, and shearing.

The nurse is caring for a newborn immediately after delivery. Which of the following actions would be appropriate? A. Perform APGAR assessment at five and ten minutes [27%] B. Suctions the nose then the mouth [33%] C. Administer RhoGAM intramuscularly [2%] D. Place the infant skin to skin with a parent [37%]

Explanation Choice D is correct. A newborn is at risk of cold stress during the first few hours of post intrauterine life. The nurse should dry the newborn thoroughly and, place the newborn skin-to-skin with a parent. Choice A is incorrect. The APGAR assessment is completed at one and five minutes. Choice B is incorrect. If suctioning is indicated, the nurse should suction the mouth then the nose, not the nose then the mouth. Choice C is incorrect. RhoGAM is indicated for Rh-negative mothers - not newborns. Medications administered to a newborn include intramuscular Vitamin K and erythromycin eye ointment. Additional Info Additional information: When caring for a newborn, the nurse should perform an APGAR assessment at one and five minutes. The higher the score, the more stable the newborn. Scores 7 to 10 are reassuring. The assessment evaluates a newborn's color, heart rate, reflexes, muscle tone, and respiration. Suctioning is only performed if indicated. If it is indicated, the correct sequence for suctioning a newborn is the mouth, then the nose.

The ICU nurse assesses a comatose patient with a known lesion to the medulla. Which breathing pattern would the nurse expect to assess? A. Cheyne-Stokes [42%] B. Apneustic breathing [22%] C. Central neurogenic hyperventilation [19%] D. Cluster breathing [17%]

Explanation Choice D is correct. Cluster breathing is associated with lesions of the medulla or lower pons. This breathing pattern is characterized by clusters of breaths with irregular pauses in between. Choice A is incorrect. Cheyne-Stokes is associated with bilateral hemispheric disease or metabolic brain dysfunction and commonly occurs at the end of life. This breathing pattern is associated with cycles of hyperventilation and apnea. Choice B is incorrect. Apneustic breathing is associated with lesions of the mid or lower pons. This breathing pattern is characterized by a prolonged inspiratory phase or pauses alternating with expiratory pauses. Choice C is incorrect. Central neurogenic hyperventilation is associated with lesions of the brainstem between the lower midbrain and upper pons. This breathing pattern is characterized by sustained, regular, rapid, and deep breathing.

The client is admitted to the ER for atrial fibrillation. The physician on duty decides to cardiovert the patient in order to reverse the dysrhythmia and the nurse commences to prepare the patient for the procedure. All of the following are accurate preparations, except: A. Obtain informed consent. [5%] B. Maintain NPO and acquire a patent IV line. [5%] C. Give the client a brief explanation of the procedure and what he will be expecting. [8%] D. Have another nurse hold the client down when doing the procedure. [82%]

Explanation Choice D is correct. In electrical cardioversion, an electric shock is given to the patient to restore his heart back to normal. The staff should not touch the patient in order to avoid electrocution. Choice A is incorrect. Cardioversion is an elective procedure and requires an Informed consent signed by the patient. Choice B is incorrect. Placing the patient on NPO eliminates the patients' risk for aspiration. Having a patent IV line makes sure that IV access is available in case the patient needs any emergency IV drugs. Choice C is incorrect. The nurse should explain the procedure to the client in order to alleviate the client's anxiety.

A client who is 28-weeks pregnant is admitted to the gynecology ward for induction of labor due to fetal demise. Which of the following substances will be used for the effacement of the client's cervix? A. Normal saline solution [2%] B. Oxytocin IV [72%] C. Amniotomy [5%] D. Laminaria [21%]

Explanation Choice D is correct. Laminaria is dehydrated seaweed. It is inserted into the cervical canal, and once it absorbs the cervical secretions, it expands and aids in the effacement and dilatation of the cervix. Choice A is incorrect. Normal saline is no longer effective in effacing the cervix in mid-trimester abortions. Choice B is incorrect. Oxytocin induces uterine contractions. It does not efface and soften the cervix. Choice C is incorrect. An amniotomy is performed during labor to aid in the descent of the fetal head once work is established. However, in fetal demise, it does not help in effacing the cervix.

A 7-month-old infant on antibiotic therapy for seven days develops oral thrush. Nystatin oral drops 1 mL PO four times a day are prescribed. Which nursing consideration should be implemented when administering this medication? A. Give the medication along with water [8%] B. Give the medication through a nipple [15%] C. Give the medication with food [3%] D. Give 0.5 mL in each side of the mouth [73%]

Explanation Choice D is correct. Oral thrush is a fungal infection by the fungus, candida. For nystatin to be effective, it should come in contact with the infected area. Giving half of the dose on each side ensures that all sides of the mouth are covered in order to reach the infected area. Choice A is incorrect. Giving the medication with water decreases the chances of the drug actually penetrating the infected area; thus, decrease its effectiveness. Choice B is incorrect. Giving the medication through a nipple decreases the chances of the drug actually penetrating the infected area; thus, decrease its effectiveness. Choice C is incorrect. Giving the medication with food decreases the chances of the drug actually penetrating the infected area; thus, decrease its effectiveness.

Which of the following are signs of decreased cardiac output in an infant with congenital heart disease? Select all that apply. A. Poor feeding [38%] B. Irritability [35%] C. Bradycardia [24%] D. Increased urine output [3%]

Explanation Choices A and B are correct. Poor feeding is often one of the first signs of decreased cardiac output in an infant. It becomes harder for the infant to breathe while feeding; they often become sweaty and pale during feedings. This is a classic sign of decreased cardiac output (Choice A). Irritability, restlessness, or lethargy are vital signs of decreased cardiac output in the infant (Choice B). Choice C is incorrect. Tachycardia, not bradycardia, would be a sign of decreased cardiac output. The body senses decreased perfusion and provides feedback to the heart to beat faster to make up for it. In doing so, the infant compensates for the decreased cardiac output for some time. Only after their body can no longer keep up will it progress to bradycardia. Choice D is incorrect. Decreased urine output would be a sign of decreased cardiac output. As the perfusion to the body lessens, blood is reserved for essential organs and the kidneys do not get as much blood flow; eventually leading to decreased urine output. NCSBN Client Need: Topic: Physiological Integrity Subtopic: Physiological Adaptation, Subtopic: Cardiovascular

While reviewing fetal monitoring strips, the labor and delivery nurse notes that the reading is nonreassuring. What features characterize a fetal monitoring strip as nonreassuring? Select all that apply. A. Fetal heart rate less than 110 beats/minute. [35%] B. Increase in variability. [16%] C. Late decelerations [36%] D. Mild variable decelerations [13%]

Explanation Choices A and C are correct. A fetal heart rate less than 110 beats/minute or greater than 160 beats/minute is nonreassuring (Choice A). Late decelerations are an ominous sign, therefore, immediate interventions should be taken to improve the fetal heart rate; they are characteristic of a nonreassuring heart rate (Choice C). Choice B is incorrect. An increase in variability is a reassuring factor. A decrease in variability would be nonreassuring. Choice D is incorrect. Mild, variable decelerations are okay, only when the variable decelerations are severe are they nonreassuring. NCSBN Client Need Topic: Physiological AdaptationSubtopic: Alterations in Body Systems

A nurse is caring for a client receiving digoxin. The client's most recent digitalis level was 2.5 ng/mL. The nurse should take which priority actions? Select all that apply. A. Withhold the client's scheduled dose. [27%] B. Administer the dose as prescribed. [2%] C. Assess the client's urinary output. [16%] D. Assess the client's most recent sodium level. [12%] E. Assess the client's heart rate and rhythm. [29%] F. Call the physician and obtain an order for a 2D echocardiogram. [13%]

Explanation Choices A and E are correct. The client's digitalis level of 2.5 ng/mL is indicative of toxicity. Digoxin has a narrow therapeutic index, which means it can cause significant side effects, such as cardiac arrhythmias (e.g. bradycardia, heart block, ventricular arrhythmias), even at plasma concentrations only twice the therapeutic plasma concentration range. Normal corrective serum digoxin levels range from 0.5-2 ng/mL. A level higher than 2 ng/mL is considered toxic. The nurse is correct to withhold the scheduled dose (Choice A) and assess the client's heart rate and rhythm (Choice E) as the client is likely to be experiencing bradycardia. Choices B, C, D, and F are incorrect. It would be wrong to administer the next dose, as this would exacerbate the toxicity. An assessment of the urinary output and sodium is not relative to digitalis toxicity and is not the priority here. Calling the physician to notify regarding the toxic level is appropriate, but there is no reason to obtain a 2D echocardiogram. A 2D echocardiogram will not add any additional information at this point. Instead, an electrocardiogram must be obtained to look for any rhythm disturbances due to digoxin toxicity.

You are completing a health history of a 4-year-old male at the primary care office. When checking with his mother about milestones in fine motor development. You would expect that the 4-year-old is able to do which of the following? Select all that apply. A. Complete a puzzle with 5 or more pieces [22%] B. Copy a triangle onto a piece of paper [27%] C. Dress himself [22%] D. Use a fork to eat dinner [30%]

Explanation Choices A, B, C, and D are all correct. These are all fine motor skills that are expected in preschool-age children, who are 3 to 5 years old. Other fine motor developmental milestones include: pasting things onto paper, completing puzzles with 5 or more pieces, cutting out simple shapes with scissors, and brushing their teeth. NCSBN Client Need: Topic: Psychosocial Integrity; Subtopic: Pediatrics - Development

The nurse is caring for a client with a port. Which of the following actions would be appropriate to take? Select all that apply. A. Access the port using an aseptic technique. [30%] B. Flush the port with heparin prior to de-access. [19%] C. Access the port using a 16-gauge catheter. [10%] D. Have the patient wear a mask during the dressing change. [17%] E. Aspirate for blood return prior to medication administration. [25%]

Explanation Choices A, B, D, and E are correct. A port is a central venous line that is useful for individuals receiving chemotherapy. The nurse should utilize an aseptic technique to prevent central line-associated bloodstream infections (CLABSIs) when the port is accessed. This includes the nurse and the client wearing a mask as well as the nurse using sterile gloves. Occlusion is a common complication with a port, and prior to de-accessing, the nurse should flush heparin. Further, the client should be instructed to wear a mask to prevent contamination during dressing changes. Finally, the nurse must verify appropriate access by aspirating for blood return prior to medication administration. Choice C is incorrect. The nurse utilizes a non-coring needle to access a port. A 16-gauge catheter will be an option if a nurse starts a large-bore peripheral IV. When a port is accessed, it is accessed with a non-coring needle that is 0.5 to 2 inches, with the gauge being 19 to 22. Additional Info When caring for a client with a port, it is essential to prevent CLABSIs through meticulous hand hygiene and aseptic dressing changes. The nurse should instruct the patient to avoid getting the accessed port wet and report signs of infection such as erythema. If the port is not in use, it needs to be accessed and flushed once a month to maintain patency.

The nurse is assessing a patient with hypothyroidism. Which of the following physical assessment findings would be expected? Select all that apply. A. Decreased libido [22%] B. Bradycardia [24%] C. Heat intolerance [5%] D. Fatigue [25%] E. Constipation [24%]

Explanation Choices A, B, D, and E are correct. Hypothyroidism causes an overall slowing of metabolic processes that contribute to patients developing constipation, fatigue, weight gain, bradycardia, periorbital edema, and decreased libido. Choices C is incorrect. Heat intolerance is a manifestation associated with hyperthyroidism. Other features of hyperthyroidism include palpitations, anxiety, weight loss, dyspnea, and increase frequency in bowel movements. Additional information: Hypothyroidism is a common endocrine abnormality found in more women when compared to men. Severe hypothyroidism is termed myxedema which may lead to myxedema coma if not treated. Risk factors for hypothyroidism include female gender, family history, and certain drugs such as lithium. Additional Info

The nurse is conducting a health screening at a local health fair. Which of the following should the nurse recognize as risk factors for peptic ulcer disease (PUD)? Select all that apply. A. Prolonged ibuprofen use [22%] B. Tobacco use [20%] C. Irritable bowel syndrome [10%] D. H. pylori [22%] E. Alcohol consumption [25%]

Explanation Choices A, B, D, and E are correct. These products are gastric irritants, and overexposure to these substances may cause PUD. Prolonged NSAID use (such as ibuprofen) is commonly implicated in the causation of PUD because of their ability to inhibit COX-1 in the gastrointestinal tract. H. pylori is a gram-negative bacteria that may be acquired by an individual consuming contaminated food or water. Alcohol and tobacco usage causes more gastric acid to be discharged. When used together, the risk for PUD is substantially increased. Choice C is incorrect. IBS is a functional GI disorder that causes spasms of the large bowel. This disorder is not implicated in causing PUD. Additional Info Three ulcers make up PUD and include duodenal ulcers, gastric ulcers, and stress ulcers. Many ulcers are caused by H. pylori infection. Gastric ulcers usually develop in the antrum of the stomach near acid-secreting mucosa. Duodenal ulcers occur more often than other types. Most duodenal ulcers are present in the upper portion of the duodenum. Stress ulcers are acute gastric mucosal lesions occurring after an acute medical crisis or trauma. Treatment for PUD includes PPIs, H2 blockers, and antibiotics if the cause is H. pylori.

The nurse provides oral care to clients in the ICU. What are the benefits of providing oral care to a client in critical care? Select all that apply. A. It promotes the patient's sense of well-being. [28%] B. It prevents deterioration of the oral cavity. [29%] C. It decreases the incidence of aspiration pneumonia. [22%] D. It eliminates the need for regular flossing. [3%] E. It decreases oropharyngeal secretions. [16%] F. It compensates for an inadequate diet. [2%]

Explanation Choices A, B, and C are correct. Adequate oral hygiene is essential for promoting a client's sense of well-being and preventing deterioration of the oral cavity. Diligent oral hygiene care can also improve oral health and limit the growth of pathogens in oropharyngeal secretions, decreasing the incidence of both aspiration pneumonia and ventilator associate pneumonia (VAP). Choices D, E, and F are incorrect. Oral care does not eliminate the need for flossing. It can increase saliva production, and would not be expected to decrease oropharyngeal secretions. Mouth care has been found to improve appetite but does not compensate for poor nutrition. Additional Information: Adequate oral hygiene helps keep the airway clear, reduces the risk of developing infections, and supports client comfort and self-esteem. It is important to use a sponge cleaner or soft-bristled toothbrush with water when performing oral care. Glycerin swabs and alcohol-containing mouthwash products can alter the mouth's pH and dry out the mucous membranes, which can lead to increased bacterial growth. If ordered, diluted hydrogen peroxide solutions can be helpful in addressing crusted areas. NCLEX Category: Basic Care and Comfort Related Content: Personal Hygiene Question Type: Knowledge/Comprehension

The nurse is reviewing leadership and management concepts with a student nurse. The student nurse understands the review lesson if they made which of the following statements? Select all that apply. A. "Battery is touching with or without the intent to do harm." [22%] B. "Assault is when the nurse makes a verbal or physical threat." [30%] C. "Unintentional torts include negligence and malpractice." [23%] D. "Defamation is presenting false credentials for employment." [13%] E. "Occurrence reports reduce the liability for a negligent tort." [12%]

Explanation Choices A, B, and C are correct. Assault is a threat or an attempt to do bodily harm. This may include verbal or gestures intended to cause intimidation. Battery is defined as touching with or without the intent to do harm. Unintentional torts include negligence or malpractice. Choices D and E are incorrect. Defamation is the process of eroding an individual's reputation through verbal insults. Fraud involves deception to obtain a good or service. Occurrence reports are a necessary tool to utilize in improving care and reducing future risk; however, they do not reduce liability for a negligent tort. Additional Info Occurrence reports, also termed incident reports, should be completed whenever any action deviates from the norm. The nurse should document factually on an occurrence report and not mention it in the medical record. Malpractice is defined as harm caused by a professional individual who possesses licensure. Negligence is the failure to act. Examples of negligence include failing to report faulty equipment.

The nurse is caring for a client with systolic heart failure. Which of the following heart sounds would the nurse expect to auscultate? Select all that apply. A. S1 [26%] B. S2 [26%] C. S3 [35%] D. S4 [12%]

Explanation Choices A, B, and C are correct. S1 (Choice A) and S2 (Choice B) are normal heart sounds. These normal heart sounds would still be auscultated in a patient with heart failure. S1 is a benign heart sound caused by the closure of the mitral and tricuspid valves. S2 is a benign heart sound produced by the closure of the aortic and pulmonic valves. S3 ("ventricular gallop") and S4 ("atrial gallop") are abnormal heart sounds that can be auscultated in heart failure. Both of these heart sounds are low-pitched and best heard at the apex with the patient in the left lateral decubitus position. While S3 may sometimes be heard in healthy hearts (normal in children, pregnant women, trained athletes), S4 is almost always abnormal. While S3 is a sign of systolic heart failure, S4 is heard in diastolic heart failure. It is essential to understand these two types of congestive heart failure (CHF) before we proceed to discuss how S3 and S4 are produced. The nurse would expect to hear an S3 heart sound (Choice C) in her patient with systolic heart failure. S3 occurs after S2 with the opening of the mitral valve and a passive flow makes the sound of a large amount of blood hitting a compliant left ventricle. This large amount of blood hitting the left ventricle is because of the underlying fluid volume overload seen with systolic heart failure. Choice D is incorrect. The nurse would not expect to hear an S4 heart sound in her patient with systolic heart failure. This is seen in diastolic heart failure. S4 occurs before S1 when the atria contract to actively force blood into the left ventricle. A stiff, non-compliant left ventricle causes it. When the atrial contraction forces blood through the mitral valve, the force is increased due to resistance offered by a stiff ventricle - in this scenario, S4 is caused by the blood striking the left ventricle. S4 heart sound can also be heard in active ischemia. Please note that if a patient is having atrial fibrillation and the atria are not contracting, it is impossible to have an S4 heart sound. Know your Auscultation sites! NCSBN Client Need: Topic: Physiological Integrity, Subtopic: Physiological adaptation

The nurse in the ICU is caring for a patient receiving total parenteral nutrition (TPN). Essential nursing diagnoses on the care plan for this patient include: Select all that apply. A. Risk for fluid volume imbalance [24%] B. Risk for infection [30%] C. Risk for hyperglycemia [35%] D. Risk for gastrointestinal tract dysfunction [11%]

Explanation Choices A, B, and C are correct. This patient is at risk for fluid volume imbalance, infection, and hyperglycemia. These risks occur because of the high volume of glucose in most TPN solutions and because the nurse administers the solution through a central venous catheter. Any time the nurse delivers nutrition through an IV catheter, the patient is at risk for fluid volume imbalance. Choice D is incorrect. TPN does NOT predispose a patient to gastrointestinal tract dysfunction. However, in a case with a patient who has GI dysfunction, enteral nutrition through an NG or other tube might be contraindicated. NCSBN Client Need Topic: Pharmacological and Parenteral Therapies, Sub-topic: Total Parenteral Nutrition; Pharmacology; Gastrointestinal/Nutritional

The nurse is caring for a client with a phosphorus level of 5.3 mg/dL. The nurse identifies which of the following as possible causes of this condition? Select all that apply. A. Tumor lysis syndrome [22%] B. Hypoparathyroidism [25%] C. Hypercalcemia [16%] D. Renal failure [28%] E. Anorexia [8%]

Explanation Choices A, B, and D are correct. A is correct. This client has a phosphorus level of 5.3, which is greater than the normal 3.0-4.5 mg/dL. Tumor lysis syndrome can cause increased phosphorus levels, because when a tumor lyses the cellular contents (including phosphorus) are spilled out into the blood causing an increase in their serum levels. B is correct. Hypoparathyroidism is a cause of hyperphosphatemia. The client who experiences hypoparathyroidism has too little parathyroid hormone (PTH). PTH regulates the serum calcium concentration through its effects on the bones, kidneys, and intestines. When there is too little PTH, there are decreased calcium levels (hypocalcemia). Since calcium and phosphorus have an inverse relationship, when there are low levels of calcium there are high levels of phosphorus. Thus, hypoparathyroidism causes hyperphosphatemia. D is correct. Renal failure is a cause of hyperphosphatemia. Due to reduced kidney function, phosphorus is not able to be excreted as readily as it normally would, so increased levels of phosphorus build up in the blood causing hyperphosphatemia. Choice C is incorrect. Hypercalcemia is a cause of hypophosphatemia. This client has a phosphorus level of 5.3, which is greater than the normal 3.0-4.5 mg/dL. Phosphorus and calcium have an inverse relationship, when there are high levels of calcium there are low levels of phosphorus. Thus, hypercalcemia would cause hypophosphatemia. Choice E is incorrect. Anorexia is a cause of hypophosphatemia due to poor dietary intake. NCSBN Client Need: Topic: Lab Values, Reduction of Risk Potential; Subtopic: Potential for Alterations in Body Systems, Fluids & Electrolytes

Which of these scenarios would be considered a violation of HIPAA laws? Select all that apply. A. Locating a co-worker's address in her medical chart so you can surprise her with a birthday gift. [38%] B. Discussing discharge plans with a patient in a multi-bed recovery room with the curtain drawn around the patient's bed. [8%] C. Looking up the medical information of a friend that is not in your care but who gave you permission. [33%] D. Checking on your spouse's medical record because you are listed as her power of attorney. [21%]

Explanation Choices A, C, and D are correct. Under HIPAA rules, healthcare agencies and their employees must take steps to ensure the confidentiality of patient information and medical records. Nurses and other healthcare providers must protect the patient's right to privacy by not sharing patient information with unauthorized individuals. Also, HIPAA allows patients to see, make corrections to, and obtain copies of their medical records. Seeking out a person's personal information by reading his/her medical chart is a violation of HIPAA. Only individuals who are providing direct care to a client should access that client's medical record. One must have a medical power of attorney and a signed HIPAA release of information form permitting them to access information before it can be legally obtained. Choice B is incorrect. Discussing discharge plans in a multi-bed recovery room is considered "incidental disclosure" and is not considered a HIPAA violation. NCSBN Client Need Topic: Safe and Effective Care Environment, Subtopic: Coordinated Care, Health Insurance Portability and Accountability Act

The registered nurse (RN) is observing licensed practical/vocational nurses (LPN/VN) care for assigned patients. Which of the following actions by the LPN would require the RN to intervene? Select all that apply. A. Irrigates an indwelling catheter with warm tap water. [24%] B. Administers glargine insulin for a patient on nothing by mouth (NPO) status. [14%] C. Obtains a 12-lead electrocardiogram for a patient with hyperkalemia. [5%] D. Clamps a chest tube while the patient ambulates. [30%] E. Repositions a patient who requires log rolling by using a gait belt. [27%]

Explanation Choices A, D, and E are correct. An indwelling catheter is irrigated with sterile water or sterile normal saline. Irrigating an indwelling catheter with tap water would introduce pathogens into the bladder. A chest tube should never be clamped as it will cause a rapid increase in intrathoracic pressure, which may cause a tension pneumothorax. A patient requiring log rolling should be repositioned with more than one staff member and with a transfer sheet. A gait belt is used when a patient is ambulating. Choices B and C are incorrect. Glargine insulin is long-acting insulin with no peak. This insulin does not need to be withheld when a patient is NPO. This insulin provides a patient with basal glucose control preventing hyperglycemia. An LPN obtaining a 12-lead electrocardiogram for a patient with hyperkalemia is an appropriate action. Additional information: The scope of an LPN is as follows - the LPN may reinforce teaching, data collection, and care for patients with low acuity illnesses. The LPN may perform skills involving sterility such as tracheostomy suctioning and the insertion of an indwelling urinary catheter. Nonsterile skills that an LPN may perform include irrigation of an ostomy, medication administration (except intravenous push), and the insertion of a nasogastric tube.

The nurse is teaching a client about diabetes mellitus. Which of the following information should the nurse include? Select all that apply. A. "You will need yearly hemoglobin A1C lab tests." [10%] B. "Your diet should consist mostly of simple carbohydrates." [11%] C. "Annual visual examinations are recommended." [30%] D. "You should take more insulin before exercising." [5%] E. "Your liver enzymes will be monitored closely." [14%] F. "Check your blood sugar if you start to feel shaky." [30%]

Explanation Choices C and F are correct. A client with diabetes mellitus should be advised to have annual eye examinations because of the risk of diabetic retinopathy. Finally, the nurse should emphasize the client checks their blood glucose for any symptoms of HYPOglycemia such as palpitations, drowsiness, and feeling shaky. Choices A, B, D, and E are incorrect. Hemoglobin A1C lab tests are the standard of care for an individual with diabetes. They should be performed every 90-120 days with the goal of less than 7%. The client's diet should not contain simple carbohydrates as the bulk of these foods are processed and spike blood glucose levels. Further, the client is at risk for HYPOglycemia while exercising, and extra insulin is not recommended. Finally, a client's kidney function tests (BUN, Creatinine, GFR) - not liver function tests - will be monitored to assess for diabetic nephropathy. Additional Info An individual with diabetes mellitus will require extensive teaching by the nurse. The teaching will include blood glucose monitoring, diet, follow-up lab work, foot care, and medications. A client's hemoglobin A1C should be monitored closely as an increase indicates multiple episodes of HYPERglycemia.

The nurse is assisting with the removal of a nasogastric tube. Which of the following steps should be taken during the procedure? Select all that apply. A. Remove the air from the balloon [8%] B. Explain what is happening to the patient [26%] C. Have the client inhale deeply and hold their breath [19%] D. Pull out the nasogastric tube in one steady motion [21%] E. Gently remove the securement device from the nose [25%]

Explanation Choices B, C, D, and E are correct. B is correct. Before the nurse begins assisting with removing a nasogastric tube, she should help explain the procedure, i.e., what is happening and what the patient can expect. This is important for minimizing anxiety and ensuring the procedure goes smoothly. C is correct. The nurse should instruct the patient to inhale deeply and hold their breath as the nasogastric tube is removed. This will help the patient to better tolerate the procedure, as holding their breath will close the epiglottis over the airway so that as the tube is removed, it can easily withdraw through the esophagus and nose. D is correct. It is correct for the nurse to pull out the nasogastric tube in one steady and continuous motion. She should not pause at intervals or remove the nasogastric tube gradually since this would cause discomfort for the patient. E is correct. Before the nurse removes the nasogastric tube, the securement device will need to be removed. Depending on the facility, this could be a special tape, Tegaderm, or securement device specifically for the nasogastric tubes. Choice A is incorrect. There is no balloon filled with air at the end of a nasogastric tube. The nurse would not be able to take this action. Removing air from the balloon is necessary before removing an indwelling foley catheter, not a nasogastric tube.

Your patient has been diagnosed with Hepatitis B and is asking about the risks of the disease. Your instruction should include which of the following hazards? Select all that apply. A. Hypertension [7%] B. Cancer of the liver [24%] C. Liver cirrhosis [36%] D. Ability to transmit the infection even if the patient is asymptomatic [33%]

Explanation Choices B, C, and D are correct. Hepatitis B is an infection of the liver caused by the hepatitis B virus. It is spread through blood or body fluid from an individual with the disease to a person who is not infected. It can also be transmitted from an infected mother to a newborn at the time of birth. Unfortunately, symptoms may not occur until the liver has been damaged by liver cancer or cirrhosis. About 25% of infected people develop severe forms of liver disease. The absence of symptoms can also increase the transmission of the disease since the infected person may not know they are infected. Choice A is incorrect. Hypertension is not a known risk of hepatitis B. Common symptoms when they occur, include abdominal pain, nausea, vomiting, fever, unusually dark urine, light-colored bowel movements, and jaundice. NCSBN Client Need Topic: Health Promotion and Maintenance, Sub-Topic: Health Promotion/Disease Prevention, Safety/Infection Control

Which of the following would not be a normal change during late pregnancy? Select All That Apply. A. Waddling gait [8%] B. Sudden edema [27%] C. Vaginal bleeding [33%] D. Dark cloudy urine [31%]

Explanation Choices B, C, and D are correct. Sudden edema is abnormal and may indicate preeclampsia (Choice B). Vaginal bleeding (more than scant spotting) is never healthy in pregnancy before the start of labor (Choice C). Dark cloudy urine is abnormal and suggests infection or renal impairment (Choice D). Choice A is incorrect. Increased levels of relaxin loosen the cartilage between the pelvic bones, which results in the characteristic "waddling" walk of the third trimester. This is a healthy change during pregnancy. NCSBN Client Need Topic: Physiological Integrity, Subtopic: Reduction of Risk Potential; Pregnant Women

When making patient care assignments, the nurse delegates care activities to nursing assistive personnel [NAP]. What factors must the nurse consider? Select all that apply. A. Patient gender and ethnicity [4%] B. Complexity of the tasks [27%] C. Knowledge and skills of the NAP [34%] D. Scope of practice for the NAP [35%]

Explanation Choices B, C, and D are correct. When delegating patient care activities to nurse assistive personnel (NAP), the RN must be aware of patient needs, the complexity of the tasks to be assigned, the knowledge/skills of the individual NAP, and which jobs are appropriate to delegate according to the scope of practice for NAPs. Choice A is incorrect. Patient gender and ethnicity are not primary concerns. Bloom's Taxonomy: Applying

Which procedures necessitate the use of surgical asepsis techniques? Select all that apply. A. Intramuscular medication administration [5%] B. Central line intravenous medication administration [28%] C. Wearing gloves in the operating room [30%] D. Neonatal bathing [3%] E. Foley catheter insertion [31%] F. Emptying a urinary drainage bag [3%]

Explanation Choices B, C, and E are correct. Surgical asepsis is used when managing central line intravenous medication administration, when wearing sterile gloves in the operating room, and when inserting an indwelling Foley catheter. Asepsis is the freedom from disease-causing microorganisms. To decrease the possibility of transferring microbes from one place to another, an aseptic technique is used. The two basic types of asepsis are medical and surgical. Medical asepsis includes all practices intended to confine a specific microorganism to a particular area, limiting the number, growth, and transmission of microorganisms. In medical asepsis, objects are referred to as clean, which means the absence of almost all organisms; or dirty (soiled, contaminated), some of which may be capable of causing infection. Surgical asepsis, or sterile technique, refers to those practices that keep an area or object free of all microorganisms; it includes practices that destroy all microorganisms and spores (microscopic dormant structures formed by some pathogens that are very hardy and often survive common cleaning techniques). Surgical asepsis is used for all procedures involving the sterile areas of the body. Sepsis is the condition in which acute organ dysfunction occurs secondary to infection. Choices A, D, and F are incorrect. Medical asepsis, or clean technique, is used to administer an intramuscular injection, bathing a neonate, and emptying a urinary drainage bag. NCSBN Client Need Topic: Safe and Effective Care Environment, Subtopic: Safety and Infection Control, Types of Asepsis

The characteristics of the crisis include which of the following? Select all that apply. A. A prolonged period of time occurs before the actual anticipated crisis. [7%] B. Crises result from anticipated life-threatening events. [14%] C. A crisis results from a rapid and unanticipated life threatening event. [32%] D. Crises result from actual and perceived threats to the person. [24%] E. Crises can be quite brief and self-limiting in terms of their nature. [24%]

Explanation Choices C, D, and E are correct. A crisis results from a rapid and unanticipated life-threatening event. Crises can be precipitated in response to both actual and perceived threats to the person, and emergencies can be quite brief and self-limiting in terms of their nature. Choice A is incorrect. Crises are typically sudden and without appropriate time to be able to cope. Choice B is incorrect. Crises are most often unanticipated, and they can occur as the result of an actual or perceived life-threatening event.

The primary healthcare provider (PHCP) prescribes 400,000 units of penicillin G benzathine. The label on the medication reads penicillin G benzathine 300,000 units / 10 mL. The nurse prepares how many milliliters to administer the appropriate dose? Fill in the blank. Round your answer to the nearest whole number. mL

Explanation The formula of dose ordered / dose on hand x volume will be utilized to solve this problem. Divide the prescribed amount of medication by what is on hand 400,000 units / 300,000 units = 1.3 mL Next, take the amount of the medication and multiply it by the volume 1.3 mL x 10 mL = 13.3 mL Finally, take the dosage and round to the nearest whole number 13.3 mL = 13 mL

The primary healthcare provider (PHCP) prescribes 100 mL of 0.9% saline to infuse over 45 minutes. How many mL per hour will be administered to the client? Fill in the blank. Round your answer to the nearest whole number. mL/hr

Explanation To solve this problem, the formula of volume / time (hours) will be used. First, convert the minutes to hours 45 minutes / 60 minutes = 0.75 hrs Next, divide the prescribed total volume by the infusion time 100 mL / 0.75 hours = 133.33 Finally, take the mL/hour and round to the nearest whole number 133.33 = 133 mL/hr Additional Info 0.9% saline is an isotonic solution utilized for simple dehydration.

The nurse is preparing to suction a tracheostomy tube. Place the following actions in the correct order for this procedure: Perform hand hygiene and identify the patient. Hyperoxygenate the patient. Insert the suction catheter without suctioning to the pre-measured depth. Apply intermittent suction and rotate the suction catheter as it is removed from the tracheostomy. Replace the cap, mist collar, oxygen mask, or other apparatus to the tracheostomy and monitor the patient to ensure they return to baseline.

Perform hand hygiene and identify the patient. Hyperoxygenate the patient. Insert the suction catheter without suctioning to the pre-measured depth. Apply intermittent suction and rotate the suction catheter as it is removed from the tracheostomy. Replace the cap, mist collar, oxygen mask, or other apparatus to the tracheostomy and monitor the patient to ensure they return to baseline. Explanation Correct ordered sequence: E, A, C, B, D First, the nurse will perform hand hygiene and identify the patient. Second, hyper-oxygenate the patient to prepare them for the procedure and prevent desaturations. Third, insert the suction catheter without suctioning to the pre-measured depth. The nurse should be sure to not allow the suction catheter to touch anywhere other than the inside of the tracheostomy. It is recommended to insert the suction catheter with the nondominant hand. Fourth, apply intermittent suction and rotate the suction catheter as you remove it from the tracheostomy. The nurse should ensure the suctioning does not exceed 10 seconds. Lastly fifth, replace the cap, mist collar, oxygen mask, or other apparatus to the tracheostomy and monitor the patient to ensure they return to baseline. NCSBN Client Need Topic: Reduction of Risk Potential; Subtopic: Potential for Complications of Diagnostic Tests/Treatments/Procedures, Respiratory

Arrange Maslow's hierarchy of needs from the highest to the least priority. Physiological needs Safety Love and belonging Esteem Self-actualization

Physiological needs Safety Love and belonging Esteem Self-actualization Explanation In needs theories, human needs are ranked on an ascending scale according to how essential the requirements are for survival. One of the most renowned needs theorists, Abraham Maslow, lists human necessities on five levels. The highest priority needs are placed at the bottom of the pyramid. The correct order from highest to least priority ( from the bottom to the top of the pyramid) is: Physiological needs - Needs such as air, food, water, shelter, rest, sleep, activity, and temperature maintenance are crucial for survival. These are the highest priority and are at the bottom of Maslow's pyramid. Safety - The need for security has both physical and psychological aspects. The person needs to feel safe, both in the physical environment and in relationships. Love and belonging - The third level of needs includes giving and receiving affection, attaining a place in a group, and maintaining the feeling of belonging. Esteem - The fourth level of needs encompasses esteem for oneself (dignity, achievement, mastery, freedom) and the need to be accepted and valued by others (e.g., social status, prestige). Esteem needs to constitute one of the critical stages in achieving contentment or self-actualization. Self-actualization - When the need for self-esteem is satisfied, the individual strives for self-actualization, the innate need to develop one's maximum potential and realize one's abilities/qualities. This is a higher-level need on Maslow's pyramid; therefore, it is a lower priority. It is represented at the top of the pyramid.


Ensembles d'études connexes

Philosophy: A Christian Introduction - Ch. 7 "The Nature of Reality"

View Set

MN Property & Casualty State Exam (2023)

View Set

Passive Voice: Affirmative Sentences (Change from Active to Passive)

View Set

2.9 Given a scenario, use appropriate networking tools. 15

View Set

Musculoskeletal System Review - NPTE

View Set

Capitals - Australia and New Zealand

View Set

A+ Ch. 20 Virtualization, Linux, and Mac OS X

View Set